%indent
* 複素数の偏角の正接 [#d96b0980]
;,一般に、複素数$$ \:z $ = $ \:i y + x $$に関して、その偏角$$ \theta $$の正接は虚部と実部の比で表せる。
#ceq(e)
   $$ \tan $ \theta $$
   $$ = $ \tan $ \arg $ \:z $$
   $$ = $ \ffd{\operatorname{Im}\:z}{\operatorname{Re}\:z} $ = $ \ffd{y}{x} $$
#ceq(d)

;,また、複素数積が偏角の和になるため、自乗の偏角が偏角の倍数になる。
#ceq(e)
   $$ n\theta $ = $ n $ \arg $ \:z $ = $ \arg $ \:z^n $$
#ceq(d)

;,そこで、$$ t $ = $ \ffd{y}{x} $$、$$ \:w $ = $ \ffd{\:z}{x} $ = $ \:i $ t $ + $ 1 $$と置けば、
;,$$ \tan $ \theta $ = $ \tan $ \arg $ \:z $ = $ \tan \arg $ \:w $ = $ t $$。
;,$$ \tan $ (n\theta) $ = $ \tan $ \arg $ \:w^n $ = $ \tan $ \arg $ (\:i t + 1)^n $$。

;,以下、この関係を利用し、$$ \tan $ (n\theta) $$を二項展開として導く。

* 正接の2倍角公式 [#x6d133c3]
;,$$ \arg $ w^2 $ = $ 2\theta $$であるため、
;,$$ \:w^2 $ = $ ( $ \:it + 1 $ )^2 $$
  $$ = $ (\:it)^2 $ + $ 2 $ (\:it) $ + $ 1 $$
  $$ = $ \:i $ ( $ 2t $ ) $ + $ ( $ 1 $ - $ t^2 $ ) $$から
;,$$ \tan $ (2\theta) $ = $ \ffd{2t}{1-t^2} $ = $ \ffd{2\tan\theta}{1-\tan^2\theta} $$が導ける。

* 正接の3倍角公式 [#l78087d7]
;,同様に、$$ \arg $ w^3 $ = $ 3\theta $$であるため、
;,$$ \:w^3 $ = $ ( $ \:it + 1 $ )^3 $$
  $$ = $ (\:it)^3 $ + $ 3 $ (\:it)^2 $ + $ 3 $ (\:it) $ + $ 1 $$
  $$ = $ \:i $ ( $ 3t - t^3 $ ) $ + $ ( $ 1 $ - $ 3t^2 $ ) $$になるため、
;,$$ \tan $ (3\theta) $ = $ \ffd{3t - t^3}{1-3t^2} $ = $ \ffd{3\tan\theta - \tan^3\theta}{1-3\tan^2\theta} $$が導ける。

* 正接の$$ n $$倍角公式 [#o47a73bf]
;,一般に、$$ \arg $ w^n $ = $ n\theta $$であるため、
;,    $$ \:w^n $ = $ ( $ \:it + 1 $ )^2 $$
      $$ = $ \sum_{0 \le k \le n} $ {}_n C_{k} $ (\:it)^k $ \cancelto{\scriptstyle1}{(1)^{n-k}}\;\;\;\; $$
      $$ = $ \sum_{0 \le k \le n} $ \:i^k $ {}_n C_{k} $ t^k $$
;,  $$ = $ \sum_{0 \le k \le n}^{k \% 4 = 0} $ \cancelto{\scriptstyle   1}{\:i^0} {}_n C_{k} $ t^k $$ 
      $$ + $ \sum_{0 \le k \le n}^{k \% 4 = 1} $ \cancelto{\scriptstyle \:i}{\:i^1} {}_n C_{k} $ t^k $$ 
      $$ + $ \sum_{0 \le k \le n}^{k \% 4 = 2} $ \cancelto{\scriptstyle-  1}{\:i^2} {}_n C_{k} $ t^k $$ 
      $$ + $ \sum_{0 \le k \le n}^{k \% 4 = 3} $ \cancelto{\scriptstyle-\:i}{\:i^3} {}_n C_{k} $ t^k $$ 
;,  $$ = $ \:i $ \Big( $ \sum_{0 \le k \le n}^{k \% 4 = 1} $ {}_n C_{k} $ t^k $ - $ \sum_{0 \le k \le n}^{k \% 4 = 3} $ {}_n C_{k} $ t^k $ \Big) $$
;,  $$ + $ \;\; $ \Big( $ \sum_{0 \le k \le n}^{k \% 4 = 0} $ {}_n C_{k} $ t^k $ - $ \sum_{0 \le k \le n}^{k \% 4 = 2} $ {}_n C_{k} $ t^k $ \Big) $$
;,  $$ = $ \quad\; $ \ffd{\sum_{0 \le k \le n}^{k \% 4 = 1} {}_n C_{k} \; t^k - \sum_{0 \le k \le n}^{k \% 4 = 3} {}_n C_{k} \; t^k }{\sum_{0 \le k \le n}^{k \% 4 = 0} {}_n C_{k} \; t^k - \sum_{0 \le k \le n}^{k \% 4 = 2} {}_n C_{k} \; t^k } $$
    数学 一覧 検索 最新 バックアップ リンク元   ヘルプ   最終更新のRSS